wykazanie nierówności

Własności funkcji trygonometrycznych i cyklometrycznych. Tożsamości. RÓWNANIA I NIERÓWNOŚCI.
klimat
Użytkownik
Użytkownik
Posty: 117
Rejestracja: 13 paź 2017, o 08:24
Płeć: Mężczyzna
Lokalizacja: Tu
Podziękował: 42 razy

wykazanie nierówności

Post autor: klimat »

Niech \(\displaystyle{ x \in \left( 0; \frac{ \pi }{2} \right)}\) oraz \(\displaystyle{ x \ge \sin x}\) . Pokaż że
\(\displaystyle{ \frac{1}{x^3} - \frac{\cos x}{\sin^3 x} >0}\) .
janusz47
Użytkownik
Użytkownik
Posty: 7917
Rejestracja: 18 mar 2009, o 16:24
Płeć: Mężczyzna
Podziękował: 30 razy
Pomógł: 1671 razy

wykazanie nierówności

Post autor: janusz47 »

Można zauważyć, że dla \(\displaystyle{ x\in \left(0, \frac{\pi}{2}\right),\ \cos(x)>0}\) i przy założeniu \(\displaystyle{ x\geq \sin(x)}\) tym bardziej prawdziwa jest nierówność:

\(\displaystyle{ \frac{1}{x^3} - \frac{\cos(x)}{x^3} > 0}\) ,

która równoważna jest nierównośc:i \(\displaystyle{ 1- \cos(x) >0}\) , spełnionej dla każdego \(\displaystyle{ x\in \left(0, \frac{\pi}{2}\right) .}\)

c.b.d.o.
Awatar użytkownika
timon92
Użytkownik
Użytkownik
Posty: 1657
Rejestracja: 6 paź 2008, o 16:47
Płeć: Mężczyzna
Lokalizacja: Katowice
Podziękował: 7 razy
Pomógł: 472 razy

wykazanie nierówności

Post autor: timon92 »

janusz47, z nierówności \(\displaystyle{ \frac{1}{x^3} - \frac{\cos x}{x^3} > 0}\) nie można wnioskować, że \(\displaystyle{ \frac{1}{x^3} - \frac{\cos x}{\sin^3 x} >0}\), bo dla dowolnego \(\displaystyle{ 0<x<\frac \pi 2}\) mamy \(\displaystyle{ \frac{1}{x^3} - \frac{\cos x}{x^3}> \frac{1}{x^3} - \frac{\cos x}{\sin^3 x}}\)
janusz47
Użytkownik
Użytkownik
Posty: 7917
Rejestracja: 18 mar 2009, o 16:24
Płeć: Mężczyzna
Podziękował: 30 razy
Pomógł: 1671 razy

wykazanie nierówności

Post autor: janusz47 »

Wnioskowanie jest odwrotne.

Kosinus maleje (licznik maleje) w przedziale \(\displaystyle{ \left( 0, \frac{\pi}{2}\right)}\) i mianownik rośnie \(\displaystyle{ x \geq \sin(x)}\), więc cały ułamek maleje - nierówność powinna być spełniona.

Ponadto licznik pierwszego ułamka \(\displaystyle{ 1}\) jest większy od licznika drugiego ułamka.

Jeśli jest inaczej kol. timon 92 to proszę wyprowadź mnie z błędu.
PoweredDragon
Użytkownik
Użytkownik
Posty: 817
Rejestracja: 19 lis 2016, o 23:48
Płeć: Mężczyzna
wiek: 21
Lokalizacja: Polska
Podziękował: 3 razy
Pomógł: 115 razy

wykazanie nierówności

Post autor: PoweredDragon »

Chodzi o to, że piszesz, że wnioskowanie jest odwrotne, a nie jest.
Z nierówności:

A: \(\displaystyle{ \frac{1}{x^3} - \frac{\cos x}{\sin^3 x} >0}\) wynika B: \(\displaystyle{ \frac{1}{x^3} - \frac{\cos x}{x^3} >0}\) , a ty napisałeś, że skoro nierówność A jest prawdziwa, to tym bardziej B, a my musimy wykazać nierówność A, którą wykorzystałeś dla dowodu słabszej nierówności, który jest zbędny.
Ostatnio zmieniony 26 gru 2017, o 21:45 przez PoweredDragon, łącznie zmieniany 1 raz.
Awatar użytkownika
timon92
Użytkownik
Użytkownik
Posty: 1657
Rejestracja: 6 paź 2008, o 16:47
Płeć: Mężczyzna
Lokalizacja: Katowice
Podziękował: 7 razy
Pomógł: 472 razy

wykazanie nierówności

Post autor: timon92 »

błąd polega na tym, że z nierówności \(\displaystyle{ A>0}\) chcesz wnioskować, że \(\displaystyle{ B>0}\) podczas gdy \(\displaystyle{ A>B}\)

u nas \(\displaystyle{ A=\frac{1}{x^3} - \frac{\cos x}{x^3}}\) oraz \(\displaystyle{ B=\frac{1}{x^3} - \frac{\cos x}{\sin^3 x}}\) i prosto sprawdza się, że \(\displaystyle{ A>B}\) (redukuje się to do \(\displaystyle{ x>\sin x}\))
a4karo
Użytkownik
Użytkownik
Posty: 22206
Rejestracja: 15 maja 2011, o 20:55
Płeć: Mężczyzna
Lokalizacja: Bydgoszcz
Podziękował: 38 razy
Pomógł: 3754 razy

wykazanie nierówności

Post autor: a4karo »

Mamy \(\displaystyle{ \frac{d^2}{dx^2}\frac{\sin x}{\cos^{1/3}x}=\frac{4\sin^3x}{9\cos^{7/3}x}>0}\) .

Funkcja \(\displaystyle{ f(x)=\frac{\sin x}{\cos^{1/3}x}}\) jest wypukłą, więc jej iloraz różnicowy
\(\displaystyle{ \frac{f(x)-f(0)}{x-0}=\frac{\sin x}{x\cos^{1/3}x}}\) rośnie.

Teza wynika z faktu, że \(\displaystyle{ \lim_{x\to 0_+}\frac{\sin x}{x\cos^{1/3}x}=1}\) .
janusz47
Użytkownik
Użytkownik
Posty: 7917
Rejestracja: 18 mar 2009, o 16:24
Płeć: Mężczyzna
Podziękował: 30 razy
Pomógł: 1671 razy

wykazanie nierówności

Post autor: janusz47 »

Oryginalne wykazanie nierówności przez Pana a4karo , tylko czy jej adresat jest zaawansowany w rachunku granic i pochodnych?
PoweredDragon
Użytkownik
Użytkownik
Posty: 817
Rejestracja: 19 lis 2016, o 23:48
Płeć: Mężczyzna
wiek: 21
Lokalizacja: Polska
Podziękował: 3 razy
Pomógł: 115 razy

wykazanie nierówności

Post autor: PoweredDragon »

janusz47 jak na razie nikt nie przedstawił innego dowodu, więc autor będzie musiał sobie poradzić. Bez pochodnych ciężko byłoby raczej to zrobić.
Awatar użytkownika
timon92
Użytkownik
Użytkownik
Posty: 1657
Rejestracja: 6 paź 2008, o 16:47
Płeć: Mężczyzna
Lokalizacja: Katowice
Podziękował: 7 razy
Pomógł: 472 razy

wykazanie nierówności

Post autor: timon92 »

a czy Ty januszu47 jesteś zaawansowany w elementarnej logice?

czy wiesz już, dlaczego warunki \(\displaystyle{ a>0}\) i \(\displaystyle{ a>b}\) nie implikują \(\displaystyle{ b>0}\)?
a4karo
Użytkownik
Użytkownik
Posty: 22206
Rejestracja: 15 maja 2011, o 20:55
Płeć: Mężczyzna
Lokalizacja: Bydgoszcz
Podziękował: 38 razy
Pomógł: 3754 razy

wykazanie nierówności

Post autor: a4karo »

janusz47 pisze:Wnioskowanie jest odwrotne.

Kosinus maleje (licznik maleje) w przedziale \(\displaystyle{ \left( 0, \frac{\pi}{2}\right)}\) i mianownik rośnie \(\displaystyle{ x \geq \sin(x)}\), więc cały ułamek maleje - nierówność powinna być spełniona.

Ponadto licznik pierwszego ułamka \(\displaystyle{ 1}\) jest większy od licznika drugiego ułamka.

Jeśli jest inaczej kol. timon 92 to proszę wyprowadź mnie z błędu.
Tradycyjnie już nie masz racji.

\(\displaystyle{ x^3>\sin^3x \Rightarrow \frac{1}{x^3}<\frac{1}{\sin^3x} \Rightarrow \frac{\cos x}{x^3}<\frac{\cos x}{\sin^3x} \Rightarrow -\frac{\cos x}{x^3}>-\frac{\cos x}{\sin^3x} \Rightarrow \\ \frac{1}{x^3}-\frac{\cos x}{x^3}>\frac{1}{x^3}-\frac{\cos x}{\sin^3x}\,,}\)

ale z dodatniości lewej strony nic nie wynika.
Ostatnio zmieniony 26 gru 2017, o 23:09 przez a4karo, łącznie zmieniany 1 raz.
janusz47
Użytkownik
Użytkownik
Posty: 7917
Rejestracja: 18 mar 2009, o 16:24
Płeć: Mężczyzna
Podziękował: 30 razy
Pomógł: 1671 razy

wykazanie nierówności

Post autor: janusz47 »

Odpowiadając, na Twoje pytanie kol. Timon92 wiem.
Awatar użytkownika
Premislav
Użytkownik
Użytkownik
Posty: 15687
Rejestracja: 17 sie 2012, o 13:12
Płeć: Mężczyzna
Lokalizacja: Warszawa
Podziękował: 196 razy
Pomógł: 5220 razy

wykazanie nierówności

Post autor: Premislav »

Warunek \(\displaystyle{ x\ge \sin x}\) tak naprawdę nic nie wnosi, wszakże taka nierówność zachodzi dla dowolnego \(\displaystyle{ x}\) nieujemnego (ewentualnie jak ktoś jej nie zna, to zaoszczędzi czas na jej dowodzeniu).

Jeżeli \(\displaystyle{ x\in\left( 0, \frac \pi 2\right)}\) , to
\(\displaystyle{ \sin x \ge x-\frac{x^3}{6}\\ \frac{\sin x}{x}\ge 1-\frac{x^2}{6}}\)
(pierwsza z tych nierówności wynika ze wzoru Taylora z resztą w postaci Lagrange'a i szacowania reszty).
A więc, z uwagi na to, że \(\displaystyle{ 1-\frac{x^2}{6}>0}\) w rozważanym przedziale, wystarczy wykazać, że:
\(\displaystyle{ 1-\frac{x^2}{6}>\cos^{\frac 1 3 }(x)}\) dla \(\displaystyle{ x \in \left( 0, \frac \pi 2\right)}\) ,
a równoważnie:
\(\displaystyle{ \left( 1-\frac{x^2}{6}\right)^3>\cos x\ .}\)
Nietrudno wyliczyć, że:
\(\displaystyle{ \left( 1-\frac{x^2}{6}\right)^3=1-\frac{x^2}{2}+\frac{x^4}{12}-\frac{x^6}{216}}\) .
Ponadto (znowu wzór Taylora i szacowanie reszty) zachodzi:
\(\displaystyle{ \cos x < 1-\frac{x^2}{2}+\frac{x^4}{24}}\) dla \(\displaystyle{ x \in \left( 0, \frac \pi 2\right)}\) .
Wystarczy więc uzasadnić, iż:
\(\displaystyle{ 1-\frac{x^2}{2}+\frac{x^4}{12}-\frac{x^6}{216}\ge 1-\frac{x^2}{2}+\frac{x^4}{24}}\) w omawianym przedziale. Równoważnie:
\(\displaystyle{ \frac{x^4}{24}\ge \frac{x^6}{216}\\ 9\ge x^2}\)
co dla \(\displaystyle{ x \in \left( 0, \frac \pi 2\right)}\) oczywiście jest prawdą.

Otrzymaliśmy więc:
\(\displaystyle{ \frac{\sin^3x}{x^3} >\cos x\\ \frac{1}{x^3}-\frac{\cos x}{\sin^3 x}>0}\)
c.n.d.

No ale to mniej elegancki dowód…
Ostatnio zmieniony 27 gru 2017, o 02:43 przez SlotaWoj, łącznie zmieniany 1 raz.
Powód: Poprawa wiadomości. Kompozycja.
a4karo
Użytkownik
Użytkownik
Posty: 22206
Rejestracja: 15 maja 2011, o 20:55
Płeć: Mężczyzna
Lokalizacja: Bydgoszcz
Podziękował: 38 razy
Pomógł: 3754 razy

wykazanie nierówności

Post autor: a4karo »

No i oczywiście zamiast pochodnej musi delikwent znać wzór Taylora.
Awatar użytkownika
Premislav
Użytkownik
Użytkownik
Posty: 15687
Rejestracja: 17 sie 2012, o 13:12
Płeć: Mężczyzna
Lokalizacja: Warszawa
Podziękował: 196 razy
Pomógł: 5220 razy

wykazanie nierówności

Post autor: Premislav »

Bez pochodnych to byłaby chyba masakra (pewnie coś na tym poziomie skomplikowania: ... szereg.pdf), ale szczerze mówiąc ten ciekawy fakt o ilorazie różnicowym funkcji wypukłych znam tylko z tego forum (zresztą dlatego, że Pan o tym już parę razy pisał), mimo zdania wszystkich analiz z dobrymi ocenami (no, funkcjonalnej ze słabymi, ale to trochę inna bajka), więc pomyślałem, jak uzasadniłbym to po pierwszym semestrze studiów.

Dobra, to spróbujmy trochę inaczej:
Gdy \(\displaystyle{ x \in \left( 0, \frac \pi 2\right)}\) , to
\(\displaystyle{ \frac{\sin^3 x}{\cos x} = \frac{\sin x(1-\cos^2 x)}{\cos x}=\tg x-\sin x \cos x=\sin x\left( \frac{1}{\cos x}-\cos x\right)=\\=\left( \int_{0}^{x} \cos(t) \,\dd t \right)\left( \int_{0}^{x}\left( \frac{\sin t}{\cos^2 t}+\sin t \right)\,\dd t \right) \ge \left( \int_{0}^{x}\left(\sqrt{ \frac{\sin t}{\cos t} }+ \sqrt{\sin t\cos t}\right)\,\dd t \right)^2 \ (*)}\)
na mocy nierówności Schwarza.
Ponadto funkcja \(\displaystyle{ f(t)=1+\cos t}\) jest nieujemna i malejąca w przedziale \(\displaystyle{ [0,x]}\) dla dowolnego\(\displaystyle{ x \in \left( 0, \frac \pi 2\right)}\) , natomiast \(\displaystyle{ g(t)= \sqrt{\tg t}}\) jest całkowalna w przedziale \(\displaystyle{ [0,x]}\) dla dowolnego\(\displaystyle{ x \in \left( 0, \frac \pi 2\right)}\) .
Zatem na mocy II twierdzenia o wartości średniej dla całek mamy:
\(\displaystyle{ \int_{0}^{x}\left(\sqrt{ \frac{\sin t}{\cos t} }+ \sqrt{\sin t\cos t}\right)\,\dd t=2 \int_{0}^{c_x}\sqrt{\tg t}\,\dd t+(1+\cos x) \int_{c_x}^{x}\sqrt{\tg t}\,\dd t}\)
gdzie \(\displaystyle{ c_x}\) jest pewnym punktem pośrednim między \(\displaystyle{ 0}\) a \(\displaystyle{ x}\) .
To w dość prosty sposób daje nam dowód tezy, gdy \(\displaystyle{ x \in \left( 0, \frac \pi 3\right)}\) wówczas mamy:
\(\displaystyle{ \cos x>\frac 1 2\\ 1+\cos x>\frac 3 2 \ (**)}\)
Ponadto korzystając ze znanej nierówności \(\displaystyle{ \tg t> t}\) dla \(\displaystyle{ t \in \left( 0, \frac \pi 2\right)}\) (ta akurat ma elementarny, geometryczny dowód) i z monotoniczności całki:
\(\displaystyle{ 2 \int_{0}^{c_x}\sqrt{\tg t}\,\dd t+(1+\cos x) \int_{c_x}^{x}\sqrt{\tg t}\,\dd t\ge
2 \int_{0}^{c_x} \sqrt{t}\,\dd t+(1+\cos x) \int_{c_x}^{x}\sqrt{t}\,\dd t \ge \\ \ge (1+\cos x) \int_{0}^{x} \sqrt{t}\,\dd t=(1+\cos x)\cdot \frac 2 3 x^{\frac 3 2}}\)

i teraz korzystając z \(\displaystyle{ (**)}\) , dla \(\displaystyle{ x \in \left( 0, \frac \pi 3\right)}\) otrzymujemy \(\displaystyle{ (1+\cos x)\cdot \frac 2 3 x^{\frac 3 2}\ge x^{\frac 3 2}}\) ,
czyli \(\displaystyle{ \int_{0}^{x}\left(\sqrt{ \frac{\sin t}{\cos t} }+ \sqrt{\sin t\cos t}\right)\,\dd t \ge x^{\frac 3 2}}\) dla \(\displaystyle{ x \in \left( 0, \frac \pi 3\right)}\) ,
co po podniesieniu do kwadratu i wstawieniu do \(\displaystyle{ (*)}\) daje nam dowód tezy w przypadku \(\displaystyle{ x \in \left( 0, \frac \pi 3\right)}\) .
Słowa powyżej nie ma o rachunku różniczkowym, wprawdzie o całkowym owszem.
Niech teraz \(\displaystyle{ xinleft[ frac pi 3, frac pi 2
ight)}\)
.
Hmm… Dupa biskupa, chyba się zmęczyłem, bo przez pół godziny pisałem bzdury, które skasowałem, ale wtedy nawet jakieś dość grube szacowania powinny przejść.
Po tym, jak mamy już szacowanie \(\displaystyle{ (*)}\) dla \(\displaystyle{ x \in \left( 0, \frac \pi 2\right)}\) zauważmy, że z nierówności między średnimi i z monotoniczności całki otrzymamy:
\(\displaystyle{ \int_{0}^{x}\left(\sqrt{ \frac{\sin t}{\cos t} }+ \sqrt{\sin t\cos t}\right)\,\dd t \ge \int_{0}^{x}2\sqrt{\sin t}\,\dd t}\) ,
a teraz ze znanej \(\displaystyle{ \sin t\ge \frac 2 \pi t}\) w pierwszej ćwiartce i (ponownie) z monotoniczności całki dostaniemy:
\(\displaystyle{ \int_{0}^{x}2\sqrt{\sin t}\,\dd t \ge \int_{0}^{x} 2 \sqrt{\frac 2 \pi} \sqrt{t}\,\dd t=\frac 4 3\cdot \sqrt{\frac 2\pi }x^{\frac 3 2}}\)
i po podniesieniu do kwadratu oraz zebraniu tego wszystkiego do kupy:
\(\displaystyle{ \ldots \ge \frac{32}{9\pi}x^3>x^3}\) .
Należy tu zauważyć, że ta nierówność jest istotnie mocniejsza niż wspomniana nierówność \(\displaystyle{ \tg x>x}\) w tym samym przedziale.

No cóż, to tym bardziej proste i ładne nie jest. Pozostaje chyba podziwiać dowód a4karo.

-- 27 gru 2017, o 03:25 --

Dobra, można całkiem wywalić zastosowanie II twierdzenia o wartości średniej dla całek i od razu po dojściu do \(\displaystyle{ (*)}\) szacować ze średnich i z nierówności.
\(\displaystyle{ \sin t\ge \frac{2}{\pi} t}\). Ale już żal mi usuwać to, co pisałem o tym tw. o wartości średniej.
skrócona wersja powyższego:    
Ostatnio zmieniony 27 gru 2017, o 03:01 przez SlotaWoj, łącznie zmieniany 1 raz.
Powód: Poprawa wiadomości. Kompozycja.
ODPOWIEDZ